Difference between revisions of "1978 AHSME Problems/Problem 30"

(Solution)
 
m (See also)
(2 intermediate revisions by 2 users not shown)
Line 1: Line 1:
 +
== Problem 30 ==
 +
 +
In a tennis tournament, <math>n</math> women and <math>2n</math> men play, and each player plays exactly one match with every other player.
 +
If there are no ties and the ratio of the number of matches won by women to the number of matches won by men is <math>7/5</math>, then <math>n</math> equals
 +
 +
<math>\textbf{(A) }2\qquad
 +
\textbf{(B) }4\qquad
 +
\textbf{(C) }6\qquad
 +
\textbf{(D) }7\qquad
 +
\textbf{(E) }\text{none of these}  </math>
 +
 
==Solution==
 
==Solution==
  
Line 29: Line 40:
 
</cmath>
 
</cmath>
 
and solving for <math>w</math> gives <math>w = \frac{31}{4}.</math> Since <math>w</math> must be an integer, <math>n</math> cannot be <math>2.</math> It follows that <math>n = 3,</math> so the answer is (E). (When <math>n = 3,</math> solving gives <math>w = 18.</math>)
 
and solving for <math>w</math> gives <math>w = \frac{31}{4}.</math> Since <math>w</math> must be an integer, <math>n</math> cannot be <math>2.</math> It follows that <math>n = 3,</math> so the answer is (E). (When <math>n = 3,</math> solving gives <math>w = 18.</math>)
 +
 +
== See also ==
 +
{{AHSME box|year=1978|n=I|num-b=29|after=Last Problem}}
 +
{{MAA Notice}}

Revision as of 22:51, 5 September 2021

Problem 30

In a tennis tournament, $n$ women and $2n$ men play, and each player plays exactly one match with every other player. If there are no ties and the ratio of the number of matches won by women to the number of matches won by men is $7/5$, then $n$ equals

$\textbf{(A) }2\qquad \textbf{(B) }4\qquad \textbf{(C) }6\qquad \textbf{(D) }7\qquad  \textbf{(E) }\text{none of these}$

Solution

Since there are $n$ women, the number of matches between only women is $\frac{n(n-1)}{2},$ and similarly, there are $(2n - 1)n$ matches between only men. Since every woman plays every man exactly once, there are $2n\cdot n = 2n^2$ matches which are between a man and a woman. Call these $2n^2$ matches co-ed matches, and let $w$ be the number of co-ed matches won by women.

Then it follows that \[\frac 75 = \frac{\frac{n(n-1)}{2} + w}{(2n - 1)n + 2n^2 - w},\] which can be simplified to \[w = \frac{17}{8}n^2 - \frac38 n.\]

The number of matches won by women must be less than the total number of matches, so we obtain the inequality \[\frac{17}{8}n^2 - \frac38 n\leq 2n^2.\] Rearranging and factoring gives \[0\leq -n(n-3),\] and the only integers which satisfy this inequality are $n = 0,1,2,$ and $3.$

Clearly, there could not have been $0$ people in the tournament, so $n\neq 0.$ If $n = 1,$ then there would have been only one woman and two men in the tournament, in which case the woman could not have won the majority of the matches.

We can now plug $n = 2$ back into the equation \[\frac 75 = \frac{\frac{n(n-1)}{2} + w}{(2n - 1)n + 2n^2 - w},\] and solving for $w$ gives $w = \frac{31}{4}.$ Since $w$ must be an integer, $n$ cannot be $2.$ It follows that $n = 3,$ so the answer is (E). (When $n = 3,$ solving gives $w = 18.$)

See also

1978 AHSME (ProblemsAnswer KeyResources)
Preceded by
Problem 29
Followed by
Last Problem
1 2 3 4 5 6 7 8 9 10 11 12 13 14 15 16 17 18 19 20 21 22 23 24 25 26 27 28 29 30
All AHSME Problems and Solutions

The problems on this page are copyrighted by the Mathematical Association of America's American Mathematics Competitions. AMC logo.png